Talk:2021 Fall AMC 10A Problems/Problem 19

Revision as of 13:07, 4 December 2021 by MRENTHUSIASM (talk | contribs)
(diff) ← Older revision | Latest revision (diff) | Newer revision → (diff)

An asymptote diagram for my solution would be appreciated. You can take credit for the diagram!

Note: I cannot do asymptote diagrams. from MathFun1000

Edit: Thanks MRENTHUSIASM!

Yay! :D ~MSAISUHTNERM Victorious-Hedgehog.png